![[personal profile]](https://www.dreamwidth.org/img/silk/identity/user.png)
Пусть А1 ← A2 ← A3 ← … -- проективная система абелевых групп. Первый (и единственный, кроме исходного функтора) производный функтор проективного предела limn1 An проективной системы (An) можно посчитать как коядро гомоморфизма
1−shift: ∏n=1∞ An → ∏n=1∞ An.
Ядром того же гомоморфизма является собственно проективный предел limn An.
Достаточным условием зануления производного функтора limn1 An является следующее условие Миттаг-Леффлера: если для любого m ≥ 1 убывающая последовательность подгрупп im pm,n: An → Am, n ≥ m, стабилизируется при n → ∞, то limn1 An = 0.
Является ли это условие необходимым для зануления группы limn1 An ? Нет, не является. Достаточно заметить, что производный функтор проективного предела, как и сам функтор проективного предела, коммутирует с операцией бесконечного произведения проективных систем абелевых групп.
Подобрав последовательность проективных систем абелевых групп, каждая из которых удовлетворяет условию Миттаг-Леффлера, но моменты стабилизации подгрупп-образов в каждой следующей проективной системе наступают позже, чем в предыдущей, и взяв прямое произведение такой бесконечной последовательности проективных систем, можно получить проективную систему абелевых групп, в которой стабилизация подгрупп-образов отображений проекции не наступает никогда, а производный функтор проективного предела этой проективной системы равен нулю, тем не менее.
Можно ли превратить условие Миттаг-Леффлера в необходимое и достаточное? Можно.
Теорема. Пусть A1 ← A2 ← A3 ← … -- проективная система абелевых групп. Рассмотрим бесконечную прямую сумму счетного числа копий проективной системы (An) и обозначим эту проективную систему через (Bn) = ⊕ω (An). Тогда следующие три условия эквивалентны:
- проективная система (An) удовлетворяет условию Миттаг-Леффлера;
- проективная система (Bn) удовлетворяет условию Миттаг-Леффлера;
- limn1 Bn = 0.
Кто-нибудь из читающих эти строки встречал такой результат где-нибудь?
P.S. Спросил на MathOverflow: https://mathoverflow.net/questions/270762/the-mittag-leffler-condition-as-necessary-and-sufficient
1−shift: ∏n=1∞ An → ∏n=1∞ An.
Ядром того же гомоморфизма является собственно проективный предел limn An.
Достаточным условием зануления производного функтора limn1 An является следующее условие Миттаг-Леффлера: если для любого m ≥ 1 убывающая последовательность подгрупп im pm,n: An → Am, n ≥ m, стабилизируется при n → ∞, то limn1 An = 0.
Является ли это условие необходимым для зануления группы limn1 An ? Нет, не является. Достаточно заметить, что производный функтор проективного предела, как и сам функтор проективного предела, коммутирует с операцией бесконечного произведения проективных систем абелевых групп.
Подобрав последовательность проективных систем абелевых групп, каждая из которых удовлетворяет условию Миттаг-Леффлера, но моменты стабилизации подгрупп-образов в каждой следующей проективной системе наступают позже, чем в предыдущей, и взяв прямое произведение такой бесконечной последовательности проективных систем, можно получить проективную систему абелевых групп, в которой стабилизация подгрупп-образов отображений проекции не наступает никогда, а производный функтор проективного предела этой проективной системы равен нулю, тем не менее.
Можно ли превратить условие Миттаг-Леффлера в необходимое и достаточное? Можно.
Теорема. Пусть A1 ← A2 ← A3 ← … -- проективная система абелевых групп. Рассмотрим бесконечную прямую сумму счетного числа копий проективной системы (An) и обозначим эту проективную систему через (Bn) = ⊕ω (An). Тогда следующие три условия эквивалентны:
- проективная система (An) удовлетворяет условию Миттаг-Леффлера;
- проективная система (Bn) удовлетворяет условию Миттаг-Леффлера;
- limn1 Bn = 0.
Кто-нибудь из читающих эти строки встречал такой результат где-нибудь?
P.S. Спросил на MathOverflow: https://mathoverflow.net/questions/270762/the-mittag-leffler-condition-as-necessary-and-sufficient